If the student takes neither physics nor writing, then it could be true that the student also takes neither

Asnodgrass on September 17, 2014

Help!

Can you explain how to answer this question please? Thank you!

Reply
Create a free account to read and take part in forum discussions.

Already have an account? log in

Naz on September 26, 2014

The stimulus tells us that there must be at least three courses in--"Each summer school student must take at least three courses from among the following seven."

So, if P and W are out then which other two can be out?

Answer choice (A) says H and L.

If P and W and H and L are all out, that means--in order to satisfy our at least three courses in rule--we must have M and S and T in. But, rule number 2 tells u sthat if M is in, then P and T must be out. So we cannot have M and T in together.

Answer choice (B) says H and M.

If we have P and W and H and M all out, then we must have L and S and T in. This doesn't go against any of our rules. So, since this could be true, answer choice (B) is our answer.

Hope that clears things up! Please let us know if you have any other questions.